Category theory: ex. 3.3.vi from Riehl's “Category Theory in Context”












2












$begingroup$



Prove that for any small category $mathsf{A}$, the functor category $mathsf{C^A}$ again has any limits or colimits that $mathsf{C}$ does, constructed objectwise. That is, given a diagram $mathcal{D}colonmathsf{J}tomathsf{C^A}$, with $mathsf{J}$ small, show that whenever limits of the diagrams $mathsf{ev}_amathcal{D}$ exists for any $a in mathsf{A}$, then these values define the action on objects of $limmathcal{D} in mathsf{C^A}$, a limit of the diagram $F$.




A functor $mathsf{ev}_acolonmathsf{C^A}tomathsf{C}$ maps a functor $Fcolonmathsf{Ato C}$ to $F(a)$ and a natural transformation $alphacolon FRightarrow G$ to $alpha_a$.



Here's what I have done:



I used the axiom of choice to obtain a family of limits $lim(mathsf{ev}_amathcal{D})$ together with limit cones $lambda_acolonlim(mathsf{ev}_amathcal{D})Rightarrowmathsf{ev}_amathcal{D}$. I have defined a functor $Fcolonmathsf{Ato C}$ which maps each $a in mathsf{A}$ to $F(a)$ and which maps each morphism $fcolon ato b$ to the unique morphism $F(f)$ for which we have $(lambda_b)_icirc F(f) = mathcal{D}(i)(f) circ (lambda_a)_i$ for any $i in mathsf{J}$ (this construction uses the fact that $lambda_b$ is a limit cone and defines a cone $kappacolonlim(mathsf{ev}_amathcal{D})tomathsf{ev}_bmathcal{D}$ by setting $kappa_i = mathcal{D}(i)(f)circ(lambda_a)_i)$.



Next, I have defined a cone $Lambdacolon FRightarrowmathcal{D}$ so that for any $i in mathsf{J}$, $Lambda_i$ is such a natural transformation $FRightarrowmathcal{D}(i)$ for which we have the following: for any $a in mathsf{A}$, $(Lambda_i)_a = (lambda_a)_i$.



What I need is to prove that $Lambda$ is a limit cone of $mathcal{D}$. Let $Mcolon GRightarrowmathcal{D}$ be a cone. From it we can obtain a family of cones $mu_acolon G(a)Rightarrowmathsf{ev}_amathcal{D}$ so that $(mu_a)_i = (M_i)_a$. As $lambda_a$ is a limit cone, we have a family of morphism $f_acolon G(a)tolim(mathsf{ev}_amathcal{D})$ unique in the sense that for any $i in I$ we have $(lambda_a)_icirc f_a = (mu_a)_i$.



What I can't do is to prove that $(f_a)_{a in mathsf{A}}$ is a natural transformation $GRightarrow F$. Let $gcolon ato b$ be a morphism of $mathsf{A}$. If $(f_a)$ is a natural transformation, then we must have $F(g)circ f_a = f_bcirc G(g)$. I can't prove this identity. The best I can do is this:



As $M_i$ is a natural transformation $GRightarrowmathcal{D}(i)$, we must have $mathcal{D}(i)(g)circ (M_i)_a = (M_i)_b circ G(g)$, that is, we have $mathcal{D}(i)(g)circ(mu_a)_i = (mu_b)_i circ G(g)$. Thus, $(lambda_b)_i circ F(g)circ f_a = mathcal{D}(i)(g) circ (lambda_a)_i circ f_a = mathcal{D}(i)(g)circ(mu_a)_i = (mu_b)_icirc G(g) = (lambda_b)_icirc f_bcirc G(g)$.



But it doesn't help much. So, what exactly am I missing?










share|cite|improve this question











$endgroup$

















    2












    $begingroup$



    Prove that for any small category $mathsf{A}$, the functor category $mathsf{C^A}$ again has any limits or colimits that $mathsf{C}$ does, constructed objectwise. That is, given a diagram $mathcal{D}colonmathsf{J}tomathsf{C^A}$, with $mathsf{J}$ small, show that whenever limits of the diagrams $mathsf{ev}_amathcal{D}$ exists for any $a in mathsf{A}$, then these values define the action on objects of $limmathcal{D} in mathsf{C^A}$, a limit of the diagram $F$.




    A functor $mathsf{ev}_acolonmathsf{C^A}tomathsf{C}$ maps a functor $Fcolonmathsf{Ato C}$ to $F(a)$ and a natural transformation $alphacolon FRightarrow G$ to $alpha_a$.



    Here's what I have done:



    I used the axiom of choice to obtain a family of limits $lim(mathsf{ev}_amathcal{D})$ together with limit cones $lambda_acolonlim(mathsf{ev}_amathcal{D})Rightarrowmathsf{ev}_amathcal{D}$. I have defined a functor $Fcolonmathsf{Ato C}$ which maps each $a in mathsf{A}$ to $F(a)$ and which maps each morphism $fcolon ato b$ to the unique morphism $F(f)$ for which we have $(lambda_b)_icirc F(f) = mathcal{D}(i)(f) circ (lambda_a)_i$ for any $i in mathsf{J}$ (this construction uses the fact that $lambda_b$ is a limit cone and defines a cone $kappacolonlim(mathsf{ev}_amathcal{D})tomathsf{ev}_bmathcal{D}$ by setting $kappa_i = mathcal{D}(i)(f)circ(lambda_a)_i)$.



    Next, I have defined a cone $Lambdacolon FRightarrowmathcal{D}$ so that for any $i in mathsf{J}$, $Lambda_i$ is such a natural transformation $FRightarrowmathcal{D}(i)$ for which we have the following: for any $a in mathsf{A}$, $(Lambda_i)_a = (lambda_a)_i$.



    What I need is to prove that $Lambda$ is a limit cone of $mathcal{D}$. Let $Mcolon GRightarrowmathcal{D}$ be a cone. From it we can obtain a family of cones $mu_acolon G(a)Rightarrowmathsf{ev}_amathcal{D}$ so that $(mu_a)_i = (M_i)_a$. As $lambda_a$ is a limit cone, we have a family of morphism $f_acolon G(a)tolim(mathsf{ev}_amathcal{D})$ unique in the sense that for any $i in I$ we have $(lambda_a)_icirc f_a = (mu_a)_i$.



    What I can't do is to prove that $(f_a)_{a in mathsf{A}}$ is a natural transformation $GRightarrow F$. Let $gcolon ato b$ be a morphism of $mathsf{A}$. If $(f_a)$ is a natural transformation, then we must have $F(g)circ f_a = f_bcirc G(g)$. I can't prove this identity. The best I can do is this:



    As $M_i$ is a natural transformation $GRightarrowmathcal{D}(i)$, we must have $mathcal{D}(i)(g)circ (M_i)_a = (M_i)_b circ G(g)$, that is, we have $mathcal{D}(i)(g)circ(mu_a)_i = (mu_b)_i circ G(g)$. Thus, $(lambda_b)_i circ F(g)circ f_a = mathcal{D}(i)(g) circ (lambda_a)_i circ f_a = mathcal{D}(i)(g)circ(mu_a)_i = (mu_b)_icirc G(g) = (lambda_b)_icirc f_bcirc G(g)$.



    But it doesn't help much. So, what exactly am I missing?










    share|cite|improve this question











    $endgroup$















      2












      2








      2





      $begingroup$



      Prove that for any small category $mathsf{A}$, the functor category $mathsf{C^A}$ again has any limits or colimits that $mathsf{C}$ does, constructed objectwise. That is, given a diagram $mathcal{D}colonmathsf{J}tomathsf{C^A}$, with $mathsf{J}$ small, show that whenever limits of the diagrams $mathsf{ev}_amathcal{D}$ exists for any $a in mathsf{A}$, then these values define the action on objects of $limmathcal{D} in mathsf{C^A}$, a limit of the diagram $F$.




      A functor $mathsf{ev}_acolonmathsf{C^A}tomathsf{C}$ maps a functor $Fcolonmathsf{Ato C}$ to $F(a)$ and a natural transformation $alphacolon FRightarrow G$ to $alpha_a$.



      Here's what I have done:



      I used the axiom of choice to obtain a family of limits $lim(mathsf{ev}_amathcal{D})$ together with limit cones $lambda_acolonlim(mathsf{ev}_amathcal{D})Rightarrowmathsf{ev}_amathcal{D}$. I have defined a functor $Fcolonmathsf{Ato C}$ which maps each $a in mathsf{A}$ to $F(a)$ and which maps each morphism $fcolon ato b$ to the unique morphism $F(f)$ for which we have $(lambda_b)_icirc F(f) = mathcal{D}(i)(f) circ (lambda_a)_i$ for any $i in mathsf{J}$ (this construction uses the fact that $lambda_b$ is a limit cone and defines a cone $kappacolonlim(mathsf{ev}_amathcal{D})tomathsf{ev}_bmathcal{D}$ by setting $kappa_i = mathcal{D}(i)(f)circ(lambda_a)_i)$.



      Next, I have defined a cone $Lambdacolon FRightarrowmathcal{D}$ so that for any $i in mathsf{J}$, $Lambda_i$ is such a natural transformation $FRightarrowmathcal{D}(i)$ for which we have the following: for any $a in mathsf{A}$, $(Lambda_i)_a = (lambda_a)_i$.



      What I need is to prove that $Lambda$ is a limit cone of $mathcal{D}$. Let $Mcolon GRightarrowmathcal{D}$ be a cone. From it we can obtain a family of cones $mu_acolon G(a)Rightarrowmathsf{ev}_amathcal{D}$ so that $(mu_a)_i = (M_i)_a$. As $lambda_a$ is a limit cone, we have a family of morphism $f_acolon G(a)tolim(mathsf{ev}_amathcal{D})$ unique in the sense that for any $i in I$ we have $(lambda_a)_icirc f_a = (mu_a)_i$.



      What I can't do is to prove that $(f_a)_{a in mathsf{A}}$ is a natural transformation $GRightarrow F$. Let $gcolon ato b$ be a morphism of $mathsf{A}$. If $(f_a)$ is a natural transformation, then we must have $F(g)circ f_a = f_bcirc G(g)$. I can't prove this identity. The best I can do is this:



      As $M_i$ is a natural transformation $GRightarrowmathcal{D}(i)$, we must have $mathcal{D}(i)(g)circ (M_i)_a = (M_i)_b circ G(g)$, that is, we have $mathcal{D}(i)(g)circ(mu_a)_i = (mu_b)_i circ G(g)$. Thus, $(lambda_b)_i circ F(g)circ f_a = mathcal{D}(i)(g) circ (lambda_a)_i circ f_a = mathcal{D}(i)(g)circ(mu_a)_i = (mu_b)_icirc G(g) = (lambda_b)_icirc f_bcirc G(g)$.



      But it doesn't help much. So, what exactly am I missing?










      share|cite|improve this question











      $endgroup$





      Prove that for any small category $mathsf{A}$, the functor category $mathsf{C^A}$ again has any limits or colimits that $mathsf{C}$ does, constructed objectwise. That is, given a diagram $mathcal{D}colonmathsf{J}tomathsf{C^A}$, with $mathsf{J}$ small, show that whenever limits of the diagrams $mathsf{ev}_amathcal{D}$ exists for any $a in mathsf{A}$, then these values define the action on objects of $limmathcal{D} in mathsf{C^A}$, a limit of the diagram $F$.




      A functor $mathsf{ev}_acolonmathsf{C^A}tomathsf{C}$ maps a functor $Fcolonmathsf{Ato C}$ to $F(a)$ and a natural transformation $alphacolon FRightarrow G$ to $alpha_a$.



      Here's what I have done:



      I used the axiom of choice to obtain a family of limits $lim(mathsf{ev}_amathcal{D})$ together with limit cones $lambda_acolonlim(mathsf{ev}_amathcal{D})Rightarrowmathsf{ev}_amathcal{D}$. I have defined a functor $Fcolonmathsf{Ato C}$ which maps each $a in mathsf{A}$ to $F(a)$ and which maps each morphism $fcolon ato b$ to the unique morphism $F(f)$ for which we have $(lambda_b)_icirc F(f) = mathcal{D}(i)(f) circ (lambda_a)_i$ for any $i in mathsf{J}$ (this construction uses the fact that $lambda_b$ is a limit cone and defines a cone $kappacolonlim(mathsf{ev}_amathcal{D})tomathsf{ev}_bmathcal{D}$ by setting $kappa_i = mathcal{D}(i)(f)circ(lambda_a)_i)$.



      Next, I have defined a cone $Lambdacolon FRightarrowmathcal{D}$ so that for any $i in mathsf{J}$, $Lambda_i$ is such a natural transformation $FRightarrowmathcal{D}(i)$ for which we have the following: for any $a in mathsf{A}$, $(Lambda_i)_a = (lambda_a)_i$.



      What I need is to prove that $Lambda$ is a limit cone of $mathcal{D}$. Let $Mcolon GRightarrowmathcal{D}$ be a cone. From it we can obtain a family of cones $mu_acolon G(a)Rightarrowmathsf{ev}_amathcal{D}$ so that $(mu_a)_i = (M_i)_a$. As $lambda_a$ is a limit cone, we have a family of morphism $f_acolon G(a)tolim(mathsf{ev}_amathcal{D})$ unique in the sense that for any $i in I$ we have $(lambda_a)_icirc f_a = (mu_a)_i$.



      What I can't do is to prove that $(f_a)_{a in mathsf{A}}$ is a natural transformation $GRightarrow F$. Let $gcolon ato b$ be a morphism of $mathsf{A}$. If $(f_a)$ is a natural transformation, then we must have $F(g)circ f_a = f_bcirc G(g)$. I can't prove this identity. The best I can do is this:



      As $M_i$ is a natural transformation $GRightarrowmathcal{D}(i)$, we must have $mathcal{D}(i)(g)circ (M_i)_a = (M_i)_b circ G(g)$, that is, we have $mathcal{D}(i)(g)circ(mu_a)_i = (mu_b)_i circ G(g)$. Thus, $(lambda_b)_i circ F(g)circ f_a = mathcal{D}(i)(g) circ (lambda_a)_i circ f_a = mathcal{D}(i)(g)circ(mu_a)_i = (mu_b)_icirc G(g) = (lambda_b)_icirc f_bcirc G(g)$.



      But it doesn't help much. So, what exactly am I missing?







      proof-verification proof-writing category-theory limits-colimits






      share|cite|improve this question















      share|cite|improve this question













      share|cite|improve this question




      share|cite|improve this question








      edited Dec 24 '18 at 8:44









      Oskar

      3,2831819




      3,2831819










      asked Dec 23 '18 at 20:32









      Jxt921Jxt921

      1,038719




      1,038719






















          1 Answer
          1






          active

          oldest

          votes


















          1












          $begingroup$

          From the last equality you obtain that $F(g)circ f_a=f_bcirc G(g)$, because $lambda_b$ is a limiting cone.






          share|cite|improve this answer









          $endgroup$













          • $begingroup$
            But for that to be of use we have to prove that the family $(mathcal{D}(i)(g)circ(mu_a)_i)_{i in mathsf{I}}$ defines a cone from $G(a)$ to $mathsf{ev}_bmathcal{D}$, right?
            $endgroup$
            – Jxt921
            Dec 24 '18 at 12:21










          • $begingroup$
            @Jxt921 A composition of a natural transformation and a cone is always a cone.
            $endgroup$
            – Oskar
            Dec 24 '18 at 12:40












          • $begingroup$
            Yeah, I know this. What I meant is one needs to prove that $(mathcal{D}(i)(g))_{i in mathsf{J}}$ indeed defines a natural transformation. But, turns out, I have already proven a similar result earlier (it is needed to defined the functor $F$ on morphisms). Thanks.
            $endgroup$
            – Jxt921
            Dec 24 '18 at 12:49














          Your Answer








          StackExchange.ready(function() {
          var channelOptions = {
          tags: "".split(" "),
          id: "69"
          };
          initTagRenderer("".split(" "), "".split(" "), channelOptions);

          StackExchange.using("externalEditor", function() {
          // Have to fire editor after snippets, if snippets enabled
          if (StackExchange.settings.snippets.snippetsEnabled) {
          StackExchange.using("snippets", function() {
          createEditor();
          });
          }
          else {
          createEditor();
          }
          });

          function createEditor() {
          StackExchange.prepareEditor({
          heartbeatType: 'answer',
          autoActivateHeartbeat: false,
          convertImagesToLinks: true,
          noModals: true,
          showLowRepImageUploadWarning: true,
          reputationToPostImages: 10,
          bindNavPrevention: true,
          postfix: "",
          imageUploader: {
          brandingHtml: "Powered by u003ca class="icon-imgur-white" href="https://imgur.com/"u003eu003c/au003e",
          contentPolicyHtml: "User contributions licensed under u003ca href="https://creativecommons.org/licenses/by-sa/3.0/"u003ecc by-sa 3.0 with attribution requiredu003c/au003e u003ca href="https://stackoverflow.com/legal/content-policy"u003e(content policy)u003c/au003e",
          allowUrls: true
          },
          noCode: true, onDemand: true,
          discardSelector: ".discard-answer"
          ,immediatelyShowMarkdownHelp:true
          });


          }
          });














          draft saved

          draft discarded


















          StackExchange.ready(
          function () {
          StackExchange.openid.initPostLogin('.new-post-login', 'https%3a%2f%2fmath.stackexchange.com%2fquestions%2f3050680%2fcategory-theory-ex-3-3-vi-from-riehls-category-theory-in-context%23new-answer', 'question_page');
          }
          );

          Post as a guest















          Required, but never shown

























          1 Answer
          1






          active

          oldest

          votes








          1 Answer
          1






          active

          oldest

          votes









          active

          oldest

          votes






          active

          oldest

          votes









          1












          $begingroup$

          From the last equality you obtain that $F(g)circ f_a=f_bcirc G(g)$, because $lambda_b$ is a limiting cone.






          share|cite|improve this answer









          $endgroup$













          • $begingroup$
            But for that to be of use we have to prove that the family $(mathcal{D}(i)(g)circ(mu_a)_i)_{i in mathsf{I}}$ defines a cone from $G(a)$ to $mathsf{ev}_bmathcal{D}$, right?
            $endgroup$
            – Jxt921
            Dec 24 '18 at 12:21










          • $begingroup$
            @Jxt921 A composition of a natural transformation and a cone is always a cone.
            $endgroup$
            – Oskar
            Dec 24 '18 at 12:40












          • $begingroup$
            Yeah, I know this. What I meant is one needs to prove that $(mathcal{D}(i)(g))_{i in mathsf{J}}$ indeed defines a natural transformation. But, turns out, I have already proven a similar result earlier (it is needed to defined the functor $F$ on morphisms). Thanks.
            $endgroup$
            – Jxt921
            Dec 24 '18 at 12:49


















          1












          $begingroup$

          From the last equality you obtain that $F(g)circ f_a=f_bcirc G(g)$, because $lambda_b$ is a limiting cone.






          share|cite|improve this answer









          $endgroup$













          • $begingroup$
            But for that to be of use we have to prove that the family $(mathcal{D}(i)(g)circ(mu_a)_i)_{i in mathsf{I}}$ defines a cone from $G(a)$ to $mathsf{ev}_bmathcal{D}$, right?
            $endgroup$
            – Jxt921
            Dec 24 '18 at 12:21










          • $begingroup$
            @Jxt921 A composition of a natural transformation and a cone is always a cone.
            $endgroup$
            – Oskar
            Dec 24 '18 at 12:40












          • $begingroup$
            Yeah, I know this. What I meant is one needs to prove that $(mathcal{D}(i)(g))_{i in mathsf{J}}$ indeed defines a natural transformation. But, turns out, I have already proven a similar result earlier (it is needed to defined the functor $F$ on morphisms). Thanks.
            $endgroup$
            – Jxt921
            Dec 24 '18 at 12:49
















          1












          1








          1





          $begingroup$

          From the last equality you obtain that $F(g)circ f_a=f_bcirc G(g)$, because $lambda_b$ is a limiting cone.






          share|cite|improve this answer









          $endgroup$



          From the last equality you obtain that $F(g)circ f_a=f_bcirc G(g)$, because $lambda_b$ is a limiting cone.







          share|cite|improve this answer












          share|cite|improve this answer



          share|cite|improve this answer










          answered Dec 24 '18 at 8:43









          OskarOskar

          3,2831819




          3,2831819












          • $begingroup$
            But for that to be of use we have to prove that the family $(mathcal{D}(i)(g)circ(mu_a)_i)_{i in mathsf{I}}$ defines a cone from $G(a)$ to $mathsf{ev}_bmathcal{D}$, right?
            $endgroup$
            – Jxt921
            Dec 24 '18 at 12:21










          • $begingroup$
            @Jxt921 A composition of a natural transformation and a cone is always a cone.
            $endgroup$
            – Oskar
            Dec 24 '18 at 12:40












          • $begingroup$
            Yeah, I know this. What I meant is one needs to prove that $(mathcal{D}(i)(g))_{i in mathsf{J}}$ indeed defines a natural transformation. But, turns out, I have already proven a similar result earlier (it is needed to defined the functor $F$ on morphisms). Thanks.
            $endgroup$
            – Jxt921
            Dec 24 '18 at 12:49




















          • $begingroup$
            But for that to be of use we have to prove that the family $(mathcal{D}(i)(g)circ(mu_a)_i)_{i in mathsf{I}}$ defines a cone from $G(a)$ to $mathsf{ev}_bmathcal{D}$, right?
            $endgroup$
            – Jxt921
            Dec 24 '18 at 12:21










          • $begingroup$
            @Jxt921 A composition of a natural transformation and a cone is always a cone.
            $endgroup$
            – Oskar
            Dec 24 '18 at 12:40












          • $begingroup$
            Yeah, I know this. What I meant is one needs to prove that $(mathcal{D}(i)(g))_{i in mathsf{J}}$ indeed defines a natural transformation. But, turns out, I have already proven a similar result earlier (it is needed to defined the functor $F$ on morphisms). Thanks.
            $endgroup$
            – Jxt921
            Dec 24 '18 at 12:49


















          $begingroup$
          But for that to be of use we have to prove that the family $(mathcal{D}(i)(g)circ(mu_a)_i)_{i in mathsf{I}}$ defines a cone from $G(a)$ to $mathsf{ev}_bmathcal{D}$, right?
          $endgroup$
          – Jxt921
          Dec 24 '18 at 12:21




          $begingroup$
          But for that to be of use we have to prove that the family $(mathcal{D}(i)(g)circ(mu_a)_i)_{i in mathsf{I}}$ defines a cone from $G(a)$ to $mathsf{ev}_bmathcal{D}$, right?
          $endgroup$
          – Jxt921
          Dec 24 '18 at 12:21












          $begingroup$
          @Jxt921 A composition of a natural transformation and a cone is always a cone.
          $endgroup$
          – Oskar
          Dec 24 '18 at 12:40






          $begingroup$
          @Jxt921 A composition of a natural transformation and a cone is always a cone.
          $endgroup$
          – Oskar
          Dec 24 '18 at 12:40














          $begingroup$
          Yeah, I know this. What I meant is one needs to prove that $(mathcal{D}(i)(g))_{i in mathsf{J}}$ indeed defines a natural transformation. But, turns out, I have already proven a similar result earlier (it is needed to defined the functor $F$ on morphisms). Thanks.
          $endgroup$
          – Jxt921
          Dec 24 '18 at 12:49






          $begingroup$
          Yeah, I know this. What I meant is one needs to prove that $(mathcal{D}(i)(g))_{i in mathsf{J}}$ indeed defines a natural transformation. But, turns out, I have already proven a similar result earlier (it is needed to defined the functor $F$ on morphisms). Thanks.
          $endgroup$
          – Jxt921
          Dec 24 '18 at 12:49




















          draft saved

          draft discarded




















































          Thanks for contributing an answer to Mathematics Stack Exchange!


          • Please be sure to answer the question. Provide details and share your research!

          But avoid



          • Asking for help, clarification, or responding to other answers.

          • Making statements based on opinion; back them up with references or personal experience.


          Use MathJax to format equations. MathJax reference.


          To learn more, see our tips on writing great answers.




          draft saved


          draft discarded














          StackExchange.ready(
          function () {
          StackExchange.openid.initPostLogin('.new-post-login', 'https%3a%2f%2fmath.stackexchange.com%2fquestions%2f3050680%2fcategory-theory-ex-3-3-vi-from-riehls-category-theory-in-context%23new-answer', 'question_page');
          }
          );

          Post as a guest















          Required, but never shown





















































          Required, but never shown














          Required, but never shown












          Required, but never shown







          Required, but never shown

































          Required, but never shown














          Required, but never shown












          Required, but never shown







          Required, but never shown







          Popular posts from this blog

          Plaza Victoria

          In PowerPoint, is there a keyboard shortcut for bulleted / numbered list?

          How to put 3 figures in Latex with 2 figures side by side and 1 below these side by side images but in...